Dermatologic Structure & Function - Burns

¡Supera tus tareas y exámenes ahora con Quizwiz!

A patient who has an infected abdominal wound develops a temperature of 104 F (40 C). All the following interventions are included in the patients plan of care. In which order should the nurse perform the following actions?(Put a comma and a space between each answer choice [A, B, C, D]). a. Administer IV antibiotics. b. Sponge patient with cool water. c. Perform wet-to-dry dressing change. d. Administer acetaminophen (Tylenol).

ANS: A, D, B, C The first action should be to administer the antibiotic because treating the infection that has caused the fever is the most important aspect of fever management. The next priority is to lower the high fever, so the nurse should administer acetaminophen to lower the temperature set point. A cool sponge bath should be done after the acetaminophen is given to lower the temperature further. The wet-to-dry dressing change will not have an immediate impact on the infection or fever and should be done last.

A patients temperature has been 101 F (38.3 C) for several days. The patients normal caloric intake to meet nutritional needs is 2000 calories per day. Knowing that the metabolic rate increases 7% for each Fahrenheit degree above 100 in body temperature, calculate the total calories the patient should receive each day.

ANS: 2140 calories

Which patient is most appropriate for the burn unit charge nurse to assign to a registered nurse (RN) who has floated from the hospital medical unit? a.A 34-year-old patient who has a weight loss of 15% from admission and requires enteral feedings. b.A 67-year-old patient who has blebs under an autograft on the thigh and has an order for bleb aspiration c.A 46-year-old patient who has just come back to the unit after having a cultured epithelial autograft to the chest d.A 65-year-old patient who has twice-daily burn debridements and dressing changes to partial-thickness facial burns

ANS: A An RN from a medical unit would be familiar with malnutrition and with administration and evaluation of response to enteral feedings. The other patients require burn assessment and care that is more appropriate for staff who regularly care for burned patients.

The nurse notes several angiomas on the legs of a 73-year-old patient. Which action should the nurse take next? a.Assess the patient for evidence of liver disease. b.Discuss the adverse effects of sun exposure on the skin. c.Educate the patient about possible skin changes with aging. d.Suggest that the patient make an appointment with a dermatologist.

ANS: A Angiomas are a common occurrence as patients age, but they may occur with systemic problems such as liver disease. The patient may want to see a dermatologist to have the angiomas removed, but this is not the initial action by the nurse. The nurse may need to educate the patient about the effects of aging on the skin and about the effects of sun exposure, but the initial action should be further assessment.

When the nurse is assessing a 42-year-old woman, the patient states that she is using topical fluorouracil (Efudex, Fluoroplex) to treat actinic keratoses on her face. Which additional information will be most important for the nurse to obtain? a. Method of birth control the patient is using b. History of extensive sun exposure by the patient c. Length of time the patient has used the medication d. Appearance of the treated areas on the patients face

ANS: A Since fluorouracil is teratogenic, it is essential that the patient use a reliable method of birth control. The other information also will be obtained by the nurse, but lack of reliable birth control has the most potential for serious adverse medication effects.

A patient with an open leg wound has a white blood cell (WBC) count of 13, 500/L and a band count of 11%. What action should the nurse take first? a.Obtain wound cultures. b.Start antibiotic therapy. c.Redress the wound with wet-to-dry dressings. d.Continue to monitor the wound for purulent drainage.

ANS: A The increase in WBC count with the increased bands (shift to the left) indicates that the patient probably has a bacterial infection, and the nurse should obtain wound cultures. Antibiotic therapy and/or dressing changes may be started, but cultures should be done first. The nurse will continue to monitor the wound, but additional actions are needed as well.

Which information should the nurse include when teaching a patient who has just received a prescription for ciprofloxacin (Cipro) to treat a urinary tract infection? a.Use a sunscreen with a high SPF when exposed to the sun. b.Sun exposure may decrease the effectiveness of the medication. c.Photosensitivity may result in an artificial-looking tan appearance. d.Wear sunglasses to avoid eye damage while taking this medication.

ANS: A The patient should stay out of the sun. If that is not possible, teach them to wear sunscreen when taking medications that can cause photosensitivity. The other statements are not accurate.

Which information should the nurse include when teaching a patient who has just received a prescription for sulfamethoxazole and trimethoprim (Septra, Bactrim) to treat a urinary tract infection? a.Use a sunscreen with a high SPF when exposed to the sun. b.Sun exposure may decrease the effectiveness of the medication. c.Photosensitivity may result in an artificial-looking tan appearance. d.Wear sunglasses to avoid eye damage while taking sulfamethoxazole.

ANS: A The patient should wear sunscreen when taking medications that can cause photosensitivity. The other statements are not accurate.

A patient with atopic dermatitis has been using a high-potency topical corticosteroid ointment for several weeks. The nurse should assess for which adverse effect? a.Thinning of the affected skin b.Alopecia of the affected areas c.Reddish-brown discoloration of the skin d.Dryness and scaling in the areas of treatment

ANS: A Thinning of the skin indicates that atrophy, a possible adverse effect of topical corticosteroids, is occurring. The health care provider should be notified so that the medication can be changed or tapered. Alopecia, red-brown discoloration, and dryness/scaling of the skin are not adverse effects of topical corticosteroid use.

An adolescent has recently been experiencing pimples. The nurse notes several closed comedones across the patient's forehead and on the nose. The nurse will expect to teach this patient about the use of which medication? a. Benzoyl peroxide b. Topical clindamycin c. Topical erythromycin d. Topical retinoids

ANS: A Benzoyl peroxide is a first-line drug for mild to moderate acne. Other topical antibiotics and retinoids are used when first-line therapy fails.

A female patient with baldness asks a nurse about the safety and efficacy of minoxidil (Rogaine). What will the nurse tell the patient? a. Hair regrowth is most effective when baldness has developed recently. b. Minoxidil cannot be used by female patients. c. Once hair has been restored, minoxidil may be discontinued, because hair loss will stop. d. Systemic side effects, such as headaches and flushing, are common.

ANS: A Minoxidil is most effective at treating recent hair loss. It may be used in female patients. Hair loss may continue even with uninterrupted treatment. Systemic side effects are not common.

To provide protection against the full range of ultraviolet (UV) radiation, an organic sunscreen must contain which agent? a. Avobenzone b. Para-aminobenzoic acid (PABA) c. Titanium dioxide d. Zinc oxide

ANS: A Only one organic sunscreen, avobenzone, absorbs the full range of UV radiation. The other agents do not protect against the full range of UV radiation. Titanium dioxide and zinc oxide are inorganic screens.

Which of these actions should the nurse take first when a patient arrives in the emergency department with facial and chest burns caused by a house fire? a.Infuse the ordered IV solution. b.Auscultate the patients lung sounds. c.Determine the extent and depth of the burns. d.Administer the ordered opioid pain medications.

ANS: B A patient with facial and chest burns is at risk for inhalation injury, and assessment of airway and breathing is the priority. The other actions will be completed after airway management is assured.

A nurse is teaching a patient with contact dermatitis of the arms and legs about ways to decrease pruritus. Which information should the nurse include in the teaching plan (select all that apply)? a.Cool, wet cloths or dressings can be used to reduce itching. b.Take cool or tepid baths several times daily to decrease itching. c.Add oil to your bath water to aid in moisturizing the affected skin. d.Rub yourself dry with a towel after bathing to prevent skin maceration. e.Use of an over-the-counter (OTC) antihistamine can reduce scratching.

ANS: A, B, E Cool or tepid baths, cool dressings, and OTC antihistamines all help reduce pruritus and scratching. Adding oil to bath water is not recommended because of the increased risk for falls. The patient should use the towel to pat (not rub) the skin dry.

Which activities can the nurse working in the outpatient clinic delegate to a licensed practical/vocational nurse (LPN/LVN) (select all that apply)? a.Administer patch testing to a patient with allergic dermatitis. b.Interview a new patient about chronic health problems and allergies. c.Apply a sterile dressing after the health care provider excises a mole. d.Teach a patient about site care after a punch biopsy of an upper arm lesion. e.Explain potassium hydroxide testing to a patient with a superficial skin infection.

ANS: A, C Skills such as administration of patch testing and sterile dressing technique are included in LPN/LVN education and scope of practice. Obtaining a health history and patient education require more critical thinking and registered nurse (RN) level education and scope of practice.

A patient with extensive electrical burn injuries is admitted to the emergency department. Which of these prescribed interventions should the nurse implement first? a.Start two large bore IVs. b.Place on cardiac monitor. c.Apply dressings to burned areas. d.Assess for pain at contact points.

ANS: B After an electrical burn, the patient is at risk for fatal dysrhythmias and should be placed on a cardiac monitor. The other actions should be accomplished in the following order: Start two IVs, assess for pain, and apply dressings.

A 38-year-old female patient states that she is using topical fluorouracil to treat actinic keratoses on her face. Which additional assessment information will be most important for the nurse to obtain? a.History of sun exposure by the patient b.Method of birth control used by the patient c.Length of time the patient has used fluorouracil d.Appearance of the treated areas on the patients face

ANS: B Because fluorouracil is teratogenic, it is essential that the patient use a reliable method of birth control. The other information is also important for the nurse to obtain, but lack of reliable birth control has the most potential for serious adverse medication effects.

A 76-year-old patient has an open surgical wound on the abdomen that contains a creamy exudate and small areas of deep pink granulation tissue. The nurse documents the wound as a a.red wound. b.yellow wound. c.full-thickness wound. d.stage III pressure wound.

ANS: B The description is consistent with a yellow wound. A stage III pressure wound would expose subcutaneous fat. A red wound would not have any creamy colored exudate. A full-thickness wound involves subcutaneous tissue, which is not indicated in the wound description.

The nurse assesses a patients surgical wound on the first postoperative day and notes redness and warmth around the incision. Which action by the nurse is most appropriate? a.Obtain wound cultures. b.Document the assessment. c.Notify the health care provider. d.Assess the wound every 2 hours.

ANS: B The incisional redness and warmth are indicators of the normal initial (inflammatory) stage of wound healing by primary intention. The nurse should document the wound appearance and continue to monitor the wound. Notification of the health care provider, assessment every 2 hours, and obtaining wound cultures are not indicated because the healing is progressing normally.

A patient is admitted to the burn unit with burns to the head, face, and hands. Initially, wheezes are heard, but an hour later, the lung sounds are decreased and no wheezes are audible. What is the best action for the nurse to take? a.Encourage the patient to cough and auscultate the lungs again. b.Notify the health care provider and prepare for endotracheal intubation. c.Document the results and continue to monitor the patients respiratory rate. d.Reposition the patient in high-Fowlers position and reassess breath sounds.

ANS: B The patients history and clinical manifestations suggest airway edema and the health care provider should be notified immediately, so that intubation can be done rapidly. Placing the patient in a more upright position or having the patient cough will not address the problem of airway edema. Continuing to monitor is inappropriate because immediate action should occur.

A young adult patient who is in the rehabilitation phase 6 months after a severe face and neck burn tells the nurse, Im sorry that Im still alive. My life will never be normal again. Which response by the nurse is best? a.Most people recover after a burn and feel satisfied with their lives. b.Its true that your life may be different. What concerns you the most? c.It is really too early to know how much your life will be changed by the burn. d.Why do you feel that way? You will be able to adapt as your recovery progresses.

ANS: B This response acknowledges the patients feelings and asks for more assessment data that will help in developing an appropriate plan of care to assist the patient with the emotional response to the burn injury. The other statements are accurate, but do not acknowledge the anxiety and depression that the patient is expressing.

A 14-year-old patient has moderate acne that has not responded to topical drugs. The nurse will suggest that the patient and her parents discuss which treatment with the provider? a. Combination oral contraceptive medication b. Doxycycline (Vibramycin) c. Isotretinoin (Accutane) d. Spironolactone

ANS: B For moderate to severe acne, oral antibiotics are indicated. Doxycycline is a drug of first choice. Hormonal agents, such as oral contraceptive pills (OCPs), are used in female patients who are at least 15 years old. Isotretinoin is used when other treatments fail and acne is severe. Spironolactone is used when OCPs fail.

A patient with burns covering 40% total body surface area (TBSA) is in the acute phase of burn treatment. Which snack would be best for the nurse to offer to this patient? a.Bananas b.Orange gelatin c.Vanilla milkshake d.Whole grain bagel

ANS: C A patient with a burn injury needs high protein and calorie food intake, and the milkshake is the highest in these nutrients. The other choices are not as nutrient-dense as the milkshake. Gelatin is likely high in sugar. The bagel is a good carbohydrate choice, but low in protein. Bananas are a good source of potassium, but are not high in protein and calories.

A patient is admitted to the hospital with a pressure ulcer on the left buttock. The base of the wound is yellow and involves subcutaneous tissue. The nurse classifies the pressure ulcer as stage a. I. b. II. c. III. d. IV.

ANS: C A stage III pressure ulcer has full-thickness skin damage and extends into the subcutaneous tissue. A stage I pressure ulcer has intact skin with some observable damage such as redness or a boggy feel. Stage II pressure ulcers have partial-thickness skin loss. Stage IV pressure ulcers have full-thickness damage with tissue necrosis, extensive damage, or damage to bone, muscle, or supporting tissues.

The nurse instructs a patient about application of corticosteroid cream to an area of contact dermatitis on the right leg. Which patient action indicates that further teaching is needed? a.The patient takes a tepid bath before applying the cream. b.The patient spreads the cream using a downward motion. c.The patient applies a thick layer of the cream to the affected skin. d.The patient covers the area with a dressing after applying the cream.

ANS: C Creams and ointments should be applied in a thin layer to avoid wasting the medication. The other actions by the patient indicate that the teaching has been successful.

Which nursing action should the nurse delegate to nursing assistive personnel (NAP) who are assisting with the care of a patient with furunculosis? a.Applying antibiotic cream to the groin. b.Obtaining cultures from ruptured lesions. c.Evaluating the patients personal hygiene. d.Cleaning the skin with antimicrobial soap.

ANS: D Cleaning the skin is within the education and scope of practice for NAP. Administration of medication, obtaining cultures, and evaluation are higher-level skills that require the education and scope of practice of licensed nursing personnel.

A new nurse performs a dressing change on a stage II left heel pressure ulcer. Which action by the new nurse indicates a need for further teaching about pressure ulcer care? a.The new nurse uses a hydrocolloid dressing (DuoDerm) to cover the ulcer. b.The new nurse inserts a sterile cotton-tipped applicator into the pressure ulcer. c.The new nurse irrigates the pressure ulcer with sterile saline using a 30-mL syringe. d.The new nurse cleans the ulcer with a sterile dressing soaked in half-strength peroxide.

ANS: D Pressure ulcers should not be cleaned with solutions that are cytotoxic, such as hydrogen peroxide. The other actions by the new nurse are appropriate.

After the nurse determines that a patient has the following risk factors for melanoma, which risk factor should be the focus of patient teaching related to prevention? a.The patient has multiple dysplastic nevi. b.The patient is fair-skinned and has blue eyes. c.The patients mother died of a malignant melanoma. d.The patient uses a tanning booth throughout the winter.

ANS: D Since the only risk factor that the patient can change is the use of a tanning booth, the nurse should focus teaching about melanoma prevention on this factor. The other factors also will contribute to increased risk.

During assessment of the patients skin, the nurse observes a similar pattern of small, raised lesions on the left and right upper back areas. Which term should the nurse use to document these lesions? a.Confluent b.Zosteriform c.Generalized d.Symmetric

ANS: D The description of the lesions indicates that they are grouped. The other terms are inconsistent with the description of the lesions.

After an employee spills industrial acids on the arms and legs at work, what is the priority action that the occupational health nurse at the facility should take? a.Apply an alkaline solution to the affected area. b.Place cool compresses on the area of exposure. c.Cover the affected area with dry, sterile dressings. d.Flush the burned area with large amounts of water.

ANS: D With chemical burns, the initial action is to remove the chemical from contact with the skin as quickly as possible. Covering the affected area or placing cool compresses on the area will leave the chemical in contact with the skin. Application of an alkaline solution is not recommended.

When caring for a diabetic patient who had abdominal surgery one week ago, the nurse obtains these data. Which finding should be reported immediately to the health care provider? a.Blood glucose 136 mg/dl b.Oral temperature 101 F (38.3 C) c.Patient complaint of increased incisional pain d.New 5-cm separation of the proximal wound edges

ANS: D Wound separation at a week postoperatively indicates possible wound dehiscence and should be immediately reported to the health care provider. The other findings also will be reported, but do not require intervention as rapidly.

A patient with psoriasis has been using a high-potency glucocorticoid. Because of skin atrophy, the provider has ordered a switch to calcitriol (Vecta), a vitamin D3 analog. What will the nurse teach this patient? a. "Calcitriol causes severe photosensitivity." b. "Itching, erythema, and irritation are indications of an allergic reaction." c. "Systemic effects do not occur with this topical agent." d. "You may apply calcitriol to all areas of the skin except the face."

ANS: D Calcitriol may be applied twice daily to all areas except the face. It does not cause severe photosensitivity. Skin irritation does not occur. Systemic effects may occur.

A patient with actinic keratoses has received a prescription for fluorouracil (Carac). Which statement by the patient indicates understanding of this medication? a. "Healing should occur 6 weeks after beginning treatment." b. "I will apply this drug twice daily." c. "Severe inflammation is an indication for stopping treatment." d. "Tissue ulceration and necrosis are desired effects."

ANS: D Fluorouracil causes tissue disintegration, erosion, ulceration, and necrosis as part of the normal course of desired effects. Complete healing may take several months, although treatment lasts 2 to 6 weeks. Carac is applied once daily. Severe inflammation is part of the course of treatment and not an indication for discontinuing the medication.

An adolescent patient with moderate acne has begun a regimen consisting of combination clindamycin/benzoyl peroxide (BenzaClin) and tretinoin (Retin-A). Which statement by the patient indicates understanding of this medication regimen? a. "I should apply the Retin-A immediately after bathing." b. "I should apply the Retin-A twice daily." c. "I should augment this therapy with an abrasive soap." d. "I should use sunscreen every day."

ANS: D Tretinoin increases susceptibility to sunburn, so patients should be warned to apply a sunscreen and wear protective clothing. Before applying Retin-A, the skin should be washed, toweled dry, and allowed to dry fully for 15 to 30 minutes. Retin-A is applied once daily. Abrasive soaps intensify localized reactions to Retin-A and should not be used.

A 50-year-old patient asks about using tretinoin (Renova) to minimize wrinkles. What will the nurse will tell the patient? a. The drug may be discontinued once results are obtained. b. Results may be visible within a few weeks of starting therapy. c. Systemic toxicity is a common effect in patients with sensitive skin. d. The drug is not effective on coarse wrinkles or sun-damaged skin.

ANS: D Tretinoin is used to treat fine wrinkles, not coarse wrinkles, and does not repair sun-damaged skin. Treatment with Renova must continue to maintain the response to the drug. Results are not visible for up to 6 months after beginning therapy. Systemic toxicity is not common.

When taking the health history of an older adult, the nurse discovers that the patient has worked in the landscaping business for 40 years. The nurse will plan to teach the patient about how to self-assess for which clinical manifestations (select all that apply)? a.Vitiligo b.Alopecia c.Intertrigo d.Erythema e.Actinic keratosis

ANS: D, E A patient who has worked as a landscaper is at risk for skin lesions caused by sun exposure such as erythema and actinic keratosis. Vitiligo, alopecia, and intertrigo are not associated with excessive sun exposure.

In which order will the nurse take these actions when doing a dressing change for a partial-thickness burn wound on a patients chest? (Put a comma and a space between each answer choice [A, B, C, D, E].) a. Apply sterile gauze dressing. b. Document wound appearance. c. Apply silver sulfadiazine cream. d. Administer IV fentanyl (Sublimaze). e. Clean wound with saline-soaked gauze.

D, E, C, A, B Because partial-thickness burns are very painful, the nurses first action should be to administer pain medications. The wound will then be cleaned, antibacterial cream applied, and covered with a new sterile dressing. The last action should be to document the appearance of the wound.

The nurse is estimating the extent of a burn using the rule of nines for a patient who has been admitted with deep partial-thickness burns of the posterior trunk and right arm. What percentage of the patients total body surface area (TBSA) has been injured?

27% When using the rule of nines, the posterior trunk is considered to cover 18% of the patients body and each arm is 9%.

During assessment of the patients skin, the nurse observes a ring of small, raised, discrete lesions filled with serous fluid on the patients right temple. When documenting the lesions, the nurse will describe the lesions as a.grouped. b.confluent. c.zosteriform. d.generalized.

ANS: A The description of the lesions indicates that they are grouped. The other terms are inconsistent with the description of the lesions.

An 80-kg patient with burns over 30% of total body surface area (TBSA) is admitted to the burn unit. Using the Parkland formula of 4 mL/kg/%TBSA, what is the IV infusion rate (mL/hour) for lactated Ringers solution that the nurse will administer during the first 8 hours?

600 mL The Parkland formula states that patients should receive 4 mL/kg/%TBSA burned during the first 24 hours. Half of the total volume is given in the first 8 hours and then the last half is given over 16 hours: 4 80 30 = 9600 mL total volume; 9600/2 = 4800 mL in the first 8 hours; 4800 mL/8 hr = 600 mL/hr.

A 70 kg patient with burns over 30% of total body surface area (TBSA) is admitted to the burn unit. Using the Parkland formula, calculate the volume of lactated Ringers solution that the nursing staff will administer during the first 24 hours.

8400 mL The Parkland formula states that patients should receive 4 mL/kg/%TBSA burned during the first 24 hours.

When performing a skin assessment, the nurse notes several angiomas on the chest of an older patient. Which action should the nurse take next? a.Assess the patient for evidence of liver disease. b.Discuss the adverse effects of sun exposure on the skin. c.Teach the patient about possible skin changes with aging. d.Suggest that the patient make an appointment with a dermatologist.

ANS: A Angiomas are a common occurrence as patients get older, but they may occur with systemic problems such as liver disease. The patient may want to see a dermatologist to have the angiomas removed, but this is not the initial action by the nurse. The nurse may need to teach the patient about the effects of aging on the skin and about the effects of sun exposure, but the initial action should be further assessment.

The nurse working in the dermatology clinic assesses a young adult female patient who is taking isotretinoin (Accutane) to treat severe cystic acne. Which assessment finding is most indicative of a need for further questioning of the patient? a.The patient recently had an intrauterine device removed. b.The patient already has some acne scarring on her forehead. c.The patient has also used topical antibiotics to treat the acne. d.The patient has a strong family history of rheumatoid arthritis.

ANS: A Because isotretinoin is teratogenic, contraception is required for women who are using this medication. The nurse will need to determine whether the patient is using other birth control methods. More information about the other patient data may also be needed, but the other data do not indicate contraindications to isotretinoin use.

A patient in the dermatology clinic has a small, slow-growing papule with ulceration and a depression in the center of the lesion on the right cheek. The nurse will anticipate the need to a.prepare the patient for a biopsy. b.teach about the use of corticosteroid creams. c.educate the patient about use of tretinoin (Retin-A). d.discuss the need for topical application of antibiotics.

ANS: A Because the appearance of the lesion is consistent with a possible basal cell carcinoma (BCC), the appropriate treatment would be excision and biopsy. Over-the-counter (OTC) corticosteroids, topical antibiotics, and Retin-A would not be used for this lesion unless the biopsy indicated that the lesion was nonmalignant.

A patient in the dermatology clinic has a thin, scaly erythematous plaque on the right cheek. Which action should the nurse take? a.Prepare the patient for a biopsy. b.Teach about the use of corticosteroid creams. c.Explain how to apply tretinoin (Retin-A) to the face. d.Discuss the need for topical application of antibiotics.

ANS: A Because the appearance of the lesion suggests actinic keratosis or possible squamous cell carcinoma (SCC), the appropriate treatment would be excision and biopsy. Over-the-counter (OTC) corticosteroids, topical antibiotics, and Retin-A would not be used for this lesion.

Which integumentary assessment data from an older patient admitted with bacterial pneumonia is of most concern for the nurse? a.Reports a history of allergic rashes b.Scattered macular brown areas on extremities c.Skin brown and wrinkled, skin tenting on forearm d.Longitudinal nail bed ridges noted; sparse scalp hair

ANS: A Because the patient will be receiving antibiotics to treat the pneumonia, the nurse should be most concerned about her history of allergic rashes. The nurse needs to do further assessment of possible causes of the allergic rashes and whether she has ever had allergic reactions to any drugs, especially antibiotics. The assessment data in the other response would be normal for an older patient.

When admitting a patient with stage III pressure ulcers on both heels, which information obtained by the nurse will have the most impact on wound healing? a.The patient takes insulin daily. b.The patient states that the ulcers are very painful. c.The patient has had the heel ulcers for the last 6 months. d.The patient has several old incisions that have formed keloids.

ANS: A Chronic insulin use indicates diabetes, which can interfere with wound healing. The persistence of the ulcers over the last 6 months is a concern, but changes in care may be effective in promoting healing. Keloids are not disabling or painful, although the cosmetic effects may be distressing for some patients. Actions to reduce the patients pain will be implemented, but pain does not directly affect wound healing.

Which action will the nurse take when applying a wet dressing to an inflamed and pruritic area of skin on a patients ankle? a.Use a cool solution to wet the dressing. b.Change the dressing using sterile gloves. c.Soak the dressing in sterile normal saline. d.Apply the dressing from the knee to the foot.

ANS: A Cool solutions are used when wet dressings are applied to inflamed areas. Wet dressings do not require sterile technique; tap water is the most common solution used. To avoid maceration of healthy skin, wet dressings should only be applied over the affected area.

A patient who has diabetes is admitted for an exploratory laparotomy for abdominal pain. When planning interventions to promote wound healing, what is the nurses highest priority? a.Maintaining the patients blood glucose within a normal range b.Ensuring that the patient has an adequate dietary protein intake c.Giving antipyretics to keep the temperature less than 102 F (38.9 C) d.Redressing the surgical incision with a dry, sterile dressing twice daily

ANS: A Elevated blood glucose will have an impact on multiple factors involved in wound healing. Ensuring adequate nutrition also is important for the postoperative patient, but a higher priority is blood glucose control. A temperature of 102 F will not impact adversely on wound healing, although the nurse may administer antipyretics if the patient is uncomfortable. Application of a dry, sterile dressing daily may be ordered, but frequent dressing changes for a wound healing by primary intention is not necessary to promote wound healing.

To maintain adequate nutrition for a patient who has just been admitted with a 40% total body surface area (TBSA) burn injury, the nurse will plan to a.insert a feeding tube and initiate enteral feedings. b.infuse total parenteral nutrition via a central catheter. c.encourage an oral intake of at least 5000 kcal per day. d.administer multiple vitamins and minerals in the IV solution.

ANS: A Enteral feedings can usually be initiated during the emergent phase at low rates and increased over 24 to 48 hours to the goal rate. During the emergent phase, the patient will be unable to eat enough calories to meet nutritional needs and may have a paralytic ileus that prevents adequate nutrient absorption. Vitamins and minerals may be administered during the emergent phase, but these will not assist in meeting the patients caloric needs. Parenteral nutrition increases the infection risk, does not help preserve gastrointestinal function, and is not routinely used in burn patients.

Which nursing action will be included when the nurse is doing a wet-to-dry dressing change for a patients stage III sacral pressure ulcer? a.Administer the ordered PRN oral opioid 30 minutes before the dressing change. b.Soak the old dressings with sterile saline a few minutes before removing them. c.Pour sterile saline onto the new dry dressings after the wound has been packed. d.Apply antimicrobial ointment before repacking the wound with moist dressings.

ANS: A Mechanical debridement with wet-to-dry dressings is painful, and patients should receive pain medications before the dressing change begins. The new dressings are moistened with saline before being applied to the wound. Soaking the old dressings before removing them will eliminate the wound debridement that is the purpose of this type of dressing. Application of antimicrobial ointments is not indicated for a wet-to-dry dressing.

A patient arrives in the emergency department with a swollen ankle after an injury incurred while playing soccer. Which action by the nurse is appropriate? a.Elevate the ankle above heart level. b.Remove the patients shoe and sock. c.Apply a warm moist pack to the ankle. d.Assess the ankles range of motion (ROM).

ANS: A Soft tissue injuries are treated with rest, ice, compression, and elevation (RICE). Elevation of the ankle will decrease tissue swelling. Moving the ankle through the ROM will increase swelling and risk further injury. Cold packs should be applied the first 24 hours to reduce swelling. The soccer shoe does not need to be removed immediately and will help to compress the injury if it is left in place.

The charge nurse observes the following actions being taken by a new nurse on the burn unit. Which action by the new nurse would require an intervention by the charge nurse? a.The new nurse uses clean latex gloves when applying antibacterial cream to a burn wound. b.The new nurse obtains burn cultures when the patient has a temperature of 95.2 F (35.1 C). c.The new nurse administers PRN fentanyl (Sublimaze) IV to a patient 5 minutes before a dressing change. d.The new nurse calls the health care provider for a possible insulin order when a nondiabetic patients serum glucose is elevated.

ANS: A Sterile gloves should be worn when applying medications or dressings to a burn. Hypothermia is an indicator of possible sepsis, and cultures are appropriate. Nondiabetic patients may require insulin because stress and high calorie intake may lead to temporary hyperglycemia. Fentanyl peaks 5 minutes after IV administration, and should be used just before and during dressing changes for pain management.

A patient with circumferential burns of both arms develops a decrease in radial pulse strength and numbness in the fingers. Which action should the nurse take? a.Notify the health care provider. b.Monitor the pulses every 2 hours. c.Elevate both arms above heart level with pillows. d.Encourage the patient to flex and extend the fingers.

ANS: A The decrease in pulse in a patient with circumferential burns indicates decreased circulation to the arms and the need for escharotomy. Monitoring the pulses is not an adequate response to the decrease in circulation. Elevating the hands or increasing hand movement will not improve the patients circulation.

A patient with circumferential burns of both legs develops a decrease in dorsalis pedis pulse strength and numbness in the toes. Which action should the nurse take? a.Notify the health care provider. b.Monitor the pulses every 2 hours. c.Elevate both legs above heart level with pillows. d.Encourage the patient to flex and extend the toes on both feet.

ANS: A The decrease in pulse in a patient with circumferential burns indicates decreased circulation to the legs and the need for an escharotomy. Monitoring the pulses is not an adequate response to the decrease in circulation. Elevating the legs or increasing toe movement will not improve the patients circulation.

A patient has an open surgical wound on the abdomen that contains deep pink granulation tissue. How would the nurse document this wound? a.Red wound b.Yellow wound c.Full-thickness wound d.Stage III pressure ulcer

ANS: A The description is consistent with a red wound. A stage III pressure ulcer would expose subcutaneous fat. A yellow wound would have creamy colored exudate. A full-thickness wound involves subcutaneous tissue, which is not indicated in the wound description.

There is one opening in the schedule at the dermatology clinic, and 4 patients are seeking appointments today. Which patient will the nurse schedule for the available opening? a.38-year old with a 7-mm nevus on the face that has recently become darker b.62-year-old with multiple small, soft, pedunculated papules in both axillary areas c.42-year-old with complaints of itching after using topical fluorouracil on the nose d.50-year-old with concerns about skin redness after having a chemical peel 3 days ago

ANS: A The description of the lesion is consistent with possible malignant melanoma. This patient should be assessed as soon as possible by the health care provider. Itching is common after using topical fluorouracil and redness is an expected finding a few days after a chemical peel. Skin tags are common, benign lesions after midlife.

The nurse caring for a patient admitted with burns over 30% of the body surface assesses that urine output has dramatically increased. Which action by the nurse would best ensure adequate kidney function? a.Continue to monitor the urine output. b.Monitor for increased white blood cells (WBCs). c.Assess that blisters and edema have subsided. d.Prepare the patient for discharge from the burn unit.

ANS: A The patients urine output indicates that the patient is entering the acute phase of the burn injury and moving on from the emergent stage. At the end of the emergent phase, capillary permeability normalizes and the patient begins to diurese large amounts of urine with a low specific gravity. Although this may occur at about 48 hours, it may be longer in some patients. Blisters and edema begin to resolve, but this process requires more time. White blood cells may increase or decrease, based on the patients immune status and any infectious processes. The WBC count does not indicate kidney function. The patient will likely remain in the burn unit during the acute stage of burn injury.

A patient with an open abdominal wound has a complete blood cell (CBC) count and differential, which indicate an increase in white blood cells (WBCs) and a shift to the left. The nurse anticipates that the next action will be to a.obtain wound cultures. b.start antibiotic therapy. c.redress the wound with wet-to-dry dressings. d.continue to monitor the wound for purulent drainage.

ANS: A The shift to the left indicates that the patient probably has a bacterial infection, and the nurse will plan to obtain wound cultures. Antibiotic therapy and/or dressing changes may be started, but cultures should be done first. The nurse will continue to monitor the wound, but additional actions are needed as well.

The health care provider diagnoses impetigo in a patient who has crusty vesicopustular lesions on the lower face. Which instructions should the nurse include in the teaching plan? a.Clean the infected areas with soap and water. b.Apply alcohol-based cleansers on the lesions. c.Avoid use of antibiotic ointments on the lesions. d.Use petroleum jelly (Vaseline) to soften crusty areas.

ANS: A The treatment for impetigo includes softening of the crusts with warm saline soaks and then soap-and-water removal. Alcohol-based cleansers and use of petroleum jelly are not recommended for impetigo. Antibiotic ointments, such as mupirocin (Bactroban), may be applied to the lesions.

Which patient should the nurse assess first? a.A patient with smoke inhalation who has wheezes and altered mental status b.A patient with full-thickness leg burns who has a dressing change scheduled c.A patient with abdominal burns who is complaining of level 8 (0 to 10 scale) pain d.A patient with 40% total body surface area (TBSA) burns who is receiving IV fluids at 500 mL/hour

ANS: A This patient has evidence of lower airway injury and hypoxemia and should be assessed immediately to determine the need for oxygen or intubation. The other patients should also be assessed as rapidly as possible, but they do not have evidence of life-threatening complications.

An employee spills industrial acids on both arms and legs at work. What is the priority action that the occupational health nurse at the facility should take? a.Remove nonadherent clothing and watch. b.Apply an alkaline solution to the affected area. c.Place cool compresses on the area of exposure. d.Cover the affected area with dry, sterile dressings.

ANS: A With chemical burns, the initial action is to remove the chemical from contact with the skin as quickly as possible. Remove nonadherent clothing, shoes, watches, jewelry, glasses, or contact lenses (if face was exposed). Flush chemical from wound and surrounding area with copious amounts of saline solution or water. Covering the affected area or placing cool compresses on the area will leave the chemical in contact with the skin. Application of an alkaline solution is not recommended.

Which finding is most important for the nurse to communicate to the health care provider when caring for a patient who is receiving negative pressure wound therapy? a.Low serum albumin level b.Serosanguineous drainage c.Deep red and moist wound bed d.Cobblestone appearance of wound

ANS: A With negative pressure therapy, serum protein levels may decrease, which will adversely affect wound healing. The other findings are expected with wound healing.

A patient asks a nurse what a sun protection factor (SPF) of 15 indicates. The nurse will tell the patient that an SPF of 15 indicates: a. a 93% block of UVB radiation. b. half the protection of a sunscreen with an SPF of 30. c. low protection. d. it takes 15 minutes for the sun to burn.

ANS: A A sun protection factor of 15 indicates a 93% block of UVB. As the SPF increases, the increment in protection gets progressively smaller, so an SPF of 15 is not half that of an SPF of 30. Low protection is indicated by an SPF of 2 to 14. The SPF is calculated by the time required for the development of erythema in the protected region divided by the time required for development of erythema in the unprotected region.

A patient with severe psoriasis will begin taking acitretin (Soriatane). The nurse obtains a health history and learns that the patient takes a combination oral contraceptive. What will the nurse do? a. Counsel the patient to use another form of birth control along with the OCP. b. Tell the patient she may stop using contraception when the medication is withdrawn. c. Tell the patient that acitretin is safe to take during pregnancy. d. Tell the patient to report spotting to the provider so that another form of contraceptive may be ordered.

ANS: A Acitretin is contraindicated during pregnancy and can reduce the effectiveness of progestin-only oral contraceptives. Patients should be counseled to use two reliable forms of birth control when taking this drug. Patients must continue using birth control for at least 3 months after treatment has stopped. Acitretin is teratogenic. Patients may experience spotting with progestin-only contraceptives, which this woman is not taking.

Which of these nursing actions should be done first for a patient who has suffered a burn injury while working on an electrical power line? a.Obtain the blood pressure. b.Stabilize the cervical spine. c.Assess for the contact points. d.Check alertness and orientation.

ANS: B Cervical spine injuries are commonly associated with electrical burns. Therefore stabilization of the cervical spine takes precedence after airway management. The other actions also are included in the emergent care after electrical burns, but the most important action is to avoid spinal cord injury.

A patient has a circular, flat, reddened lesion about 5 cm in diameter on his ankle. To determine whether the lesion is related to blood vessel dilation, the nurse will a.elevate the patients leg. b.press firmly on the lesion. c.check the temperature of the skin around the lesion. d.palpate the dorsalis pedis and posterior tibial pulses.

ANS: B If the lesion is caused by blood vessel dilation, blanching will occur with direct pressure. The other assessments will assess circulation to the leg, but will not be helpful in determining the etiology of the lesion.

A patient reports chronic itching of the ankles and cannot keep from continuously scratching them. The nurse will plan to implement interventions to decrease the risk for a.skin atrophy. b.lichenification. c.skin varicosity. d.keloid formation.

ANS: B Lichenification is likely to occur in areas where the patient scratches the skin frequently. Scratching is not a risk factor for skin atrophy, keloid formation, and varicosities.

Through interviewing a patient who has a history of contact dermatitis, the nurse obtains this information about over-the-counter (OTC) medication use. Which finding indicates a need for patient teaching? a.The patient applies corticosteroid cream to any pruritic areas. b.The patient uses Neosporin ointment on minor cuts or abrasions. c.The patient adds oilated oatmeal (Aveeno) to the bath water every day. d.The patient takes diphenhydramine (Benadryl) at night if itching occurs.

ANS: B Neosporin can cause contact dermatitis. The other medications are being used appropriately by the patient.

A teenaged male patient who wrestles in high school is examined by the nurse in the clinic. Which assessment finding would prompt the nurse to teach the patient about the importance of not sharing headgear to prevent the spread of pediculosis? a.Ringlike rashes with red, scaly borders over the entire scalp b.Papular, wheal-like lesions with white deposits on the hair shaft c.Patchy areas of alopecia with small vesicles and excoriated areas d.Red, hivelike papules and plaques with sharply circumscribed borders

ANS: B Pediculosis is characterized by wheal-like lesions with parasites that attach eggs to the base of the hair shaft. The other descriptions are more characteristic of other types of skin disorders.

When examining a patients scalp, the nurse suspects the presence of pediculosis on finding a.ringlike rashes with red, scaly borders over the entire scalp. b.papular, wheal-like lesions with white deposits on the hair shaft. c.patchy areas of alopecia with small vesicles and excoriated areas. d.red, hivelike papules and plaques with sharply circumscribed borders.

ANS: B Pediculosis is characterized by wheal-like lesions with parasites that attach eggs to the base of the hair shaft. The other descriptions are more characteristic of other types of skin disorders.

On admission to the burn unit, a patient with an approximate 25% total body surface area (TBSA) burn has the following initial laboratory results: Hct 56%, Hb 17.2 mg/dL (172 g/L), serum K+ 4.8 mEq/L (4.8 mmol/L), and serum Na+ 135 mEq/L (135 mmol/L). Which action will the nurse anticipate taking? a.Continue to monitor the laboratory results. b.Increase the rate of the ordered IV solution. c.Type and crossmatch for a blood transfusion. d.Document the findings in the patients record.

ANS: B The patients lab data show hemoconcentration, which may lead to a decrease in blood flow to the microcirculation unless fluid intake is increased. Documentation and continuing to monitor are inadequate responses to the data. Since the hematocrit and hemoglobin are elevated, a transfusion is inappropriate, although transfusions may be needed after the emergent phase.

Which action will be included in the plan of care for a patient who has burns of the ears, head, neck, and right arm and hand? a.Place the right arm and hand flexed in a position of comfort. b.Elevate the right arm and hand on pillows and extend the fingers. c.Assist the patient to a supine position with a small pillow under the head. d.Position the patient in a side-lying position with rolled towel under the neck.

ANS: B The right hand and arm should be elevated to reduce swelling and the fingers extended to avoid flexion contractures (even though this position may not be comfortable for the patient). The patient with burns of the ears should not use a pillow since this will put pressure on the ears and may stick to the ears. Patients with neck burns should not use a pillow, since the head should be maintained in an extended position in order to avoid contractures.

A patient with deep partial-thickness and full-thickness burns of the face and chest is having the wounds treated with the open method. Which nursing action will be included in the plan of care? a.Restrict all visitors to prevent cross-contamination of wounds. b.Wear gowns, caps, masks, and gloves during all care of the patient. c.Turn the room temperature up to at least 68 F (20 C) during dressing changes. d.Administer prophylactic antibiotics to prevent bacterial colonization of wounds.

ANS: B Use of gowns, caps, masks, and gloves during all patient care will decrease the possibility of wound contamination for a patient whose burns are not covered. Restricting all visitors is not necessary and will have adverse psychosocial consequences for the patient. The room temperature should be kept at approximately 85 F for patients with open burn wounds. Systemic antibiotics are not well absorbed into deep burns because of the lack of circulation.

While the patients fullthickness burn wounds to the face are exposed, what is the best nursing action to prevent cross contamination? a.Use sterile gloves when removing old dressings. b.Wear gowns, caps, masks, and gloves during all care of the patient. c.Administer IV antibiotics to prevent bacterial colonization of wounds. d.Turn the room temperature up to at least 70 F (20 C) during dressing changes.

ANS: B Use of gowns, caps, masks, and gloves during all patient care will decrease the possibility of wound contamination for a patient whose burns are not covered. When removing contaminated dressings and washing the dirty wound, use nonsterile, disposable gloves. The room temperature should be kept at approximately 85 F for patients with open burn wounds to prevent shivering. Systemic antibiotics are not well absorbed into deep burns because of the lack of circulation.

When assessing a patient who spilled hot oil on the right leg and foot, the nurse notes that the skin is dry, pale, hard skin. The patient states that the burn is *not painful*. What term would the nurse use to document the burn depth? a.First-degree skin destruction b.Full-thickness skin destruction c.Deep partial-thickness skin destruction d.Superficial partial-thickness skin destruction

ANS: B With full-thickness skin destruction, the appearance is pale and dry or leathery and the area is painless because of the associated nerve destruction. Erythema, swelling, and blisters point to a deep partial-thickness burn. With superficial partial-thickness burns, the area is red, but no blisters are present. First-degree burns exhibit erythema, blanching, and pain.

An adolescent has begun using benzoyl peroxide lotion twice daily to treat acne. The patient reports experiencing drying and burning of the skin. What will the nurse suggest? a. Applying lotion to the skin after applying the drug b. Reducing the frequency to one application a day c. Discontinuing the medication, because this is likely an allergic reaction d. Requesting a prescription for a gel formulation of the drug

ANS: B Benzoyl peroxide may cause drying and peeling of the skin. If signs of severe local irritation occur, such as burning or blistering, the frequency of application should be reduced. Applying lotion is not indicated. These symptoms are not consistent with an allergic reaction. There is no difference in skin reactions between the gel and the lotion formulations.

A 50-year-old patient receives botulinum toxin type A (Botox) injections for the first time on her forehead and around her eyes. One week later, she calls the clinic to report that she is experiencing droopy eyelids. The nurse will tell the patient that this effect: a. is normal and will resolve in a few days. b. may persist for 3 to 6 months but will resolve. c. may progress to cause drooling and dysphagia. d. represents an adverse effect that may be permanent.

ANS: B Botox is a neurotoxin that acts on cholinergic neurons to block the release of acetylcholine. Injection into the wrong site or diffusion from the correct site into surrounding tissues can weaken muscles not intended as targets. This patient's droopy eyelids are an example of the wrong site being affected. Weakening of muscles can last 3 to 6 months but will resolve. It is incorrect to tell the patient that it is a normal effect or that it will resolve in a matter of days. Drooling and dysphagia are likely when injections are around the mouth. It is not a permanent effect.

A teenaged female patient has begun to develop acne and asks a nurse how to minimize pimple formation. What will the nurse recommend? a. Asking the provider about oral contraceptives b. Cleansing the face gently 2 to 3 times daily c. Eliminating greasy foods from the diet d. Using an abrasive agent to scrub the face

ANS: B Gentle cleansing 2 to 3 times a day can reduce surface oiliness and help minimize acne lesions. Oral contraceptives are not first-line treatment for acne. Eliminating greasy foods from the diet does not affect pimple formation. An abrasive agent is not indicated for mild acne.

A patient will begin initial treatment for severe acne. Which regimen will the nurse expect the provider to order? a. Clindamycin/benzoyl peroxide (BenzaClin) and tretinoin (Retin-A) b. Doxycycline (Vibramycin) and tretinoin (Retin-A) c. Erythromycin (Ery-Tab) and benzoyl peroxide d. Topical clindamycin and isotretinoin (Accutane)

ANS: B Oral antibiotics are used for moderate to severe acne and are usually combined with a topical retinoid. Combination clindamycin/benzoyl peroxide and tretinoin are used for mild to moderate acne. Erythromycin can be used as an oral antibiotic but would need to be combined with a topical retinoid. Isotretinoin is used for severe acne that has not responded to other treatments.

A patient is using a high-concentration keratolytic agent containing salicylic acid to remove warts. What will the nurse teach this patient? a. Peeling and drying are desired effects of this drug. b. Systemic effects may occur with this medication. c. Tinnitus is a common side effect of little concern. d. Tissue injury is unlikely at this dose.

ANS: B Salicylic acid is readily absorbed through the skin, and systemic toxicity can result. Peeling and drying are not desired effects of salicylic acid. Tinnitus is a symptom of systemic toxicity. Tissue injury is likely in any concentration above 6%.

A nurse is discussing the use of tazarotene (Tazorac) with a patient who has psoriasis. Which statement by the patient indicates a need for further teaching? a. "I should use a sunscreen when using this medication." b. "I understand the gel can cause staining of clothing." c. "I will apply this once daily in the evening." d. "I will apply the medication to dry skin."

ANS: B Tazarotene will not stain clothing. It is true that patients should use sunscreen while using this drug. It should be applied once daily in the evening to dry skin.

An infant has a severe contact diaper dermatitis. The provider orders triamcinolone acetonide (Kenalog) 0.1% cream to be applied 3 times daily. When teaching the infant's parents about this medication, the nurse will instruct them to apply: a. a thick layer and massage the cream into the skin. b. a thin layer and leave the diaper open as much as possible. c. the cream and place an occlusive dressing over the area. d. the cream and put the infant's diaper on tightly.

ANS: B Topical glucocorticoids can be absorbed systemically and cause adverse effects. To minimize systemic and local adverse effects, the medication should be applied sparingly. Parents should be taught to avoid tight-fitting diapers. The cream should be rubbed gently into the skin. Occlusive dressings increase the risk of adverse effects. Putting the diaper on tightly creates an occlusive dressing.

A patient with extensive electrical burn injuries is admitted to the emergency department. Which prescribed intervention should the nurse implement first? a.Assess oral temperature. b.Check a potassium level. c.Place on cardiac monitor. d.Assess for pain at contact points.

ANS: C After an electrical burn, the patient is at risk for fatal dysrhythmias and should be placed on a cardiac monitor. Assessing the oral temperature is not as important as assessing for cardiac dysrhythmias. Checking the potassium level is important. However, it will take time before the laboratory results are back. The first intervention is to place the patient on a cardiac monitor and assess for dysrhythmias, so that they can be treated if occurring. A decreased or increased potassium level will alert the nurse to the possibility of dysrhythmias. The cardiac monitor will alert the nurse immediately of any dysrhythmias. Assessing for pain is important, but the patient can endure pain until the cardiac monitor is attached. Cardiac dysrhythmias can be lethal.

A patient has just arrived in the emergency department after an electrical burn from exposure to a high-voltage current. What is the priority nursing assessment? a.Oral temperature b.Peripheral pulses c.Extremity movement d.Pupil reaction to light

ANS: C All patients with electrical burns should be considered at risk for cervical spine injury, and assessments of extremity movement will provide baseline data. The other assessment data are also necessary but not as essential as determining the cervical spine status.

A patient in the dermatology clinic is scheduled for removal of a 15-mm multicolored and irregular mole from the upper back. The nurse will plan to teach the patient about a.shave biopsy. b.punch biopsy. c.incisional biopsy. d.excisional biopsy.

ANS: C An incisional biopsy would remove the entire mole and the tissue borders. The appearance of the mole indicates that it may be malignant; a shave biopsy would not remove the entire mole. The mole is too large to be removed with punch biopsy. Excisional biopsies are done for smaller lesions and where a good cosmetic effect is desired, such as on the face.

The nurse caring for a patient admitted with burns over 30% of the body surface will recognize that the patient has moved from the emergent to the acute phase of the burn injury when a.white blood cell levels decrease. b.blisters and edema have subsided. c.the patient has large quantities of pale urine. d.the patient has been hospitalized for 48 hours.

ANS: C At the end of the emergent phase, capillary permeability normalizes and the patient begins to diurese large amounts of urine with a low specific gravity. Although this may occur at about 48 hours, it may be longer in some patients. Blisters and edema begin to resolve, but this process requires more time. White blood cells may increase or decrease, based on the patients immune status and any infectious processes.

After receiving a change-of-shift report, which patient should the nurse assess first? a.The patient who has multiple black wounds on the feet and ankles b.The newly admitted patient with a stage IV pressure ulcer on the coccyx c.The patient who has been receiving chemotherapy and has a temperature of 102 F d.The patient who needs to be medicated with multiple analgesics before a scheduled dressing change

ANS: C Chemotherapy is an immunosuppressant. Even a low fever in an immunosuppressed patient is a sign of serious infection and should be treated immediately with cultures and rapid initiation of antibiotic therapy. The nurse should assess the other patients as soon as possible after assessing and implementing appropriate care for the immunosuppressed patient.

A dark-skinned patient has been admitted to the hospital in severe respiratory distress. To determine whether the patient is cyanotic, the nurse will a.assess the skin color of the earlobes. b.apply pressure to the palms of the hands. c.check the lips and oral mucous membranes. d.examine capillary refill time of the nail beds.

ANS: C Cyanosis in dark-skinned individuals is more easily seen in the mucous membranes. Earlobe color may change in light-skinned individuals, but this change in skin color is difficult to detect on darker skin. Application of pressure to the palms of the hands and nail bed assessment would check for adequate circulation, but not for skin color.

The nurse prepares to obtain a culture from a patient who has a possible fungal infection on the foot. Which items should the nurse gather for this procedure? a.Sterile gloves b.Patch test instruments c.Cotton-tipped applicators d.Local anesthetic, syringe, and intradermal needle

ANS: C Fungal cultures are obtained by swabbing the affected area of the skin with cotton-tipped applicators. Sterile gloves are not needed because it is not a sterile procedure. Local injection is not needed because the swabbing is not usually painful. The patch test is done to determine whether a patient is allergic to specific testing material, not for obtaining fungal specimens.

The nurse is preparing to obtain a culture from a patient who has a possible fungal infection in the groin area. Which action is appropriate? a.Apply a topical anesthetic before obtaining the culture. b.Use sterile gloves to squeeze the lesion and obtain exudate. c.Swab the infected area with a sterile cotton-tipped applicator. d.Scrape the area gently with a razor blade to obtain a specimen.

ANS: C Fungal cultures are obtained by swabbing the affected area of the skin. The other actions might be used for obtaining other types of specimens.

A patient with severe burns has crystalloid fluid replacement ordered using the Parkland formula. The initial volume of fluid to be administered in the first 24 hours is 30,000 mL. The initial rate of administration is 1875 mL/hr. After the first 8 hours, the nurse will decrease the fluid infusion rate to a.350 mL/hour. b.523 mL/hour. c.938 mL/hour. d.1250 mL/hour.

ANS: C Half of the fluid replacement using the Parkland formula is administered in the first 8 hours and the other half over the next 16 hours. In this case, the patient should receive half of the initial rate, or 938 mL/hr.

When examining a patients oral cavity, the nurse notes the presence of white lesions that resemble milk curds at the back of the throat. Which question by the nurse is appropriate at this time? a.Do you have a productive cough? b.How often do you brush your teeth? c.Are you taking any medications at present? d.Have you ever had an oral herpes infection?

ANS: C The appearance of the lesions is consistent with an oral candidiasis (thrush) infection, which can occur in patients who are taking medications such as immunosuppressants or antibiotics. Candidiasis is not associated with poor oral hygiene or lower respiratory infections. The lesions do not look like an oral herpes infection.

When assessing a patient who spilled hot oil on the right leg and foot, the nurse notes that the skin is red, swollen, and covered with large blisters. The patient states that they are *very painful*. The nurse will document the injury as a.full-thickness skin destruction. b.deep full-thickness skin destruction. c.deep partial-thickness skin destruction. d.superficial partial-thickness skin destruction.

ANS: C The erythema, swelling, and blisters point to a deep partial-thickness burn. With full-thickness skin destruction, the appearance is pale and dry or leathery and the area is painless because of the associated nerve destruction. With superficial partial-thickness burns, the area is red, but no blisters are present.

A patient is undergoing psoralen plus ultraviolet A light (PUVA) therapy for treatment of psoriasis. To minimize complications from this procedure, the nurse plans to a.cleanse the skin carefully with an antiseptic soap. b.shield any unaffected areas with lead-lined drapes. c.have the patient use protective eyewear while receiving PUVA. d.apply petroleum jelly to the areas surrounding the psoriatic lesions.

ANS: C The eyes should be shielded from UV light (UVL) during and after PUVA therapy to prevent the development of cataracts. The patient should be taught about the effects of UVL on unaffected skin, but lead-lined drapes, use of antiseptic soap, and petroleum jelly are not used to prevent skin damage.

When examining an older patient in the home, the home health nurse notices irregular patterns of bruising at different stages of healing on the patients body. Which action should the nurse take first? a.Discourage the use of throw rugs throughout the house. b.Ensure the patient has a pair of shoes with non-slip soles. c.Talk with the patient alone and ask about what caused the bruising. d.Notify the health care provider so that x-rays can be ordered as soon as possible.

ANS: C The nurse should note irregular patterns of bruising, especially in the shapes of hands or fingers, in different stages of resolution. These may be indications of other health problems or abuse, and should be further investigated. It is important that the nurse interview the patient alone because, if mistreatment is occurring, the patient may not disclose it in the presence of the person who may be the abuser. Throw rugs and shoes with slippery surfaces may contribute to falls. X-rays may be needed if the patient has fallen recently and also has complaints of pain or decreased mobility. However, the nurses first nursing action is to further assess the patient.

When assessing a new patient at the outpatient clinic, the nurse notes dry, scaly skin; thin hair; and thick, brittle nails. Which action is best for the nurse to take at this time? a.Instruct the patient about the importance of nutrition in skin heath. b.Make a referral to a podiatrist so that the nails can be safely trimmed. c.Consult with the health care provider about the need for further diagnostic testing. d.Teach the patient about using moisturizing creams and lotions to decrease dry skin.

ANS: C The patient has clinical manifestations that could be caused by systemic problems such as malnutrition or hypothyroidism, so further diagnostic evaluation is indicated. Patient teaching about nutrition, addressing the patients dry skin, and referral to a podiatrist also may be needed, but the priority is to rule out underlying disease that may be causing these manifestations.

A patient with a systemic bacterial infection feels cold and has a shaking chill. Which assessment finding will the nurse expect next? a.Skin flushing b.Muscle cramps c.Rising body temperature d.Decreasing blood pressure

ANS: C The patients complaints of feeling cold and shivering indicate that the hypothalamic set point for temperature has been increased and the temperature is increasing. Because associated peripheral vasoconstriction and sympathetic nervous system stimulation will occur, skin flushing and hypotension are not expected. Muscle cramps are not expected with chills and shivering or with a rising temperature.

A patient with a systemic bacterial infection has goose pimples, feels cold, and has a shaking chill. At this stage of the febrile response, the nurse will plan to monitor for a.skin flushing. b.muscle cramps. c.rising body temperature. d.decreasing blood pressure.

ANS: C The patients complaints of feeling cold and shivering indicate that the hypothalamic set point for temperature has been increased and the temperature is increasing. Because associated peripheral vasoconstriction and sympathetic nervous system stimulation will occur, skin flushing and hypotension are not expected. Muscle cramps are not expected with chills and shivering or with rising temperatures.

A 21-year-old patient who is in the rehabilitation phase after having deep partial-thickness face and neck burns has a nursing diagnosis of disturbed body image. Which action by the patient indicates that the problem is resolving? a.Stating that the scarring will only be temporary. b.Avoiding using a pillow to prevent neck contractures. c.Asking about how to use make-up to cover up the scars. d.Expressing sadness and anger about the scar appearance.

ANS: C The willingness to use strategies to enhance appearance is an indication that the disturbed body image is resolving. Expressing feelings about the scars indicates a willingness to discuss appearance, but not resolution of the problem. Because deep partial-thickness burns leave permanent scars, a statement that the scars are temporary indicates denial rather than resolution of the problem. Avoiding using a pillow will help prevent contractures, but it does not address the problem of disturbed body image.

A patients 4 3-cm leg wound has a 0.4 cm black area in the center of the wound surrounded by yellow-green semiliquid material. Which dressing should the nurse apply to the wound? a.Dry gauze dressing (Kerlix) b.Nonadherent dressing (Xeroform) c.Hydrocolloid dressing (DuoDerm) d.Transparent film dressing (Tegaderm)

ANS: C The wound requires debridement of the necrotic areas and absorption of the yellow-green slough. A hydrocolloid dressing such as DuoDerm would accomplish these goals. Transparent film dressings are used for red wounds or approximated surgical incisions. Dry dressings will not debride the necrotic areas. Nonadherent dressings will not absorb wound drainage or debride the wound.

The health care provider prescribes topical 5-FU for a patient with actinic keratosis on the nose. Which information will the nurse include in the patient teaching plan? a.You may develop nausea and anorexia, but good nutrition is important during treatment. b.You will need to avoid crowds because of the risk for infection caused by chemotherapy. c.The nose will develop painful, eroded areas that will take weeks before completely healing. d.5-FU is needed to shrink the lesion so that less scarring occurs once the lesion is excised.

ANS: C Topical 5-FU causes an initial reaction of erythema, itching, and erosion, which lasts 4 weeks after application of the medication is stopped. The medication is topical, so there are no systemic effects such as increased infection risk, anorexia, or nausea. Actinic keratosis is not usually treated with excision.

Which information will the nurse include when teaching a 70-year-old patient about skin care? a.Dry the skin thoroughly before applying lotions. b.Bathe and shampoo daily with soap and shampoo. c.Use warm water and a moisturizing soap when bathing. d.Use antibacterial soaps when bathing to avoid infection.

ANS: C Warm water and moisturizing soap will avoid overdrying the skin. Since older patients have dryer skin, daily bathing and shampooing are not necessary and may dry the skin unnecessarily. Antibacterial soaps are not necessary. Lotions should be applied while the skin is still damp to seal moisture in.

The nurse will plan to use wet-to-dry dressings when providing care for a patient with a.a pressure ulcer with pink granulation tissue. a.surgical incision with pink, approximated edges. b.full-thickness burn filled with dry, black material. c.wound with purulent drainage and dry brown areas.

ANS: C Wet-to-dry dressings are used when there is minimal eschar to be removed. A full-thickness wound filled with eschar will require interventions such as surgical debridement to remove the necrotic tissue. Wet-to-dry dressings are not needed on approximated surgical incisions. Wet-to-dry dressings are not used on uninfected granulating wounds because of the damage to the granulation tissue.

A patient has severe acne that has been refractory to treatment. The patient is taking tetracycline and using topical tretinoin (Retin-A) and has been applying benzoyl peroxide twice daily. The provider asks the nurse to teach this patient about isotretinoin (Accutane), which the patient will begin taking in a few weeks. The nurse will include which statement when teaching this patient about this drug? a. "Alcohol may be consumed in moderation when taking this drug." b. "Skin rash, headache, and hair loss are common with this drug." c. "Tetracycline must be discontinued before beginning the isotretinoin." d. "Two pregnancy tests are required before each monthly refill of your prescription."

ANS: C Adverse effects of isotretinoin can be increased by tetracyclines, so tetracycline must be discontinued before therapy is started. Alcohol should be avoided, since it can potentiate hypertriglyceridemia. Skin rash, headache, and hair loss are not common side effects, although they can occur. Two pregnancy tests are required at the beginning of therapy; at each refill, only one pregnancy test is required.

A child with eczema has been treated unsuccessfully with a topical glucocorticoid for a year and has skin atrophy and hypopigmentation. The nurse will suggest discussing which drug with the provider? a. Higher potency topical glucocorticoids b. Topical keratolytic agents c. Topical immunosuppressants d. Topical nonsteroidal anti-inflammatory drugs (NSAIDs)

ANS: C If topical glucocorticoids fail to treat eczema without causing skin atrophy and hypopigmentation, topical immunosuppressants may be used. Higher potency glucocorticoids will only compound the adverse effects. Topical keratolytic agents are not indicated. Topical NSAIDs are not indicated.

A nurse is teaching a group of lifeguards about safe sunning. Which statement by a lifeguard indicates understanding of the teaching? a. "Sunscreen should be applied 30 minutes before going outside." b. "I do not need sunscreen when it is cloudy outside." c. "I should reapply sunscreen after swimming." d. "UV radiation does not penetrate through water."

ANS: C Sunscreens should be reapplied after swimming and with profuse sweating. Most sunscreens should be applied at least 30 minutes before going outdoors, but some require application 2 hours before exposure. Clouds do not protect from all UV rays. UV radiation can penetrate at least several centimeters of clear water.

Ranitidine (Zantac) is prescribed for a patient who incurred extensive burn injuries 5 days ago. Which information will the nurse collect to evaluate the effectiveness of the medication? a.Bowel sounds b.Stool frequency c.Abdominal distention d.Stools for occult blood

ANS: D H2 blockers are given to prevent Curlings ulcer in the patient who has suffered burn injuries. H2 blockers do not impact on bowel sounds, stool frequency, or appetite.

When taking the health history for a patient, the nurse discovers that the patient works as a roofer. The nurse will plan to teach the patient about how to self-assess for clinical manifestations of (select all that apply) a.alopecia. b.intertrigo. c.wrinkling. d.erythema. e.actinic keratosis.

ANS: C, D, E A patient who works as a roofer is at risk for integumentary lesions caused by sun exposure such as wrinkling, erythema, and actinic keratoses. Alopecia and intertrigo are not associated with excessive sun exposure.

Which of these snacks will be best for the nurse to offer to a patient with burns covering 40% total body surface area (TBSA) who is in the acute phase of burn treatment? a.Strawberry gelatin b.Whole wheat bagel c.Chunky applesauce d.Chocolate milkshake

ANS: D A patient with a burn injury needs high protein and calorie food intake, and the milkshake is the highest in these nutrients. The other choices are not as nutrient-dense as the milkshake.

A patient who has burns on the arms, legs, and chest from a house fire has become agitated and restless 8 hours after being admitted to the hospital. Which action should the nurse take first? a.Stay at the bedside and reassure the patient. b.Administer the ordered morphine sulfate IV. c.Assess orientation and level of consciousness. d.Use pulse oximetry to check the oxygen saturation.

ANS: D Agitation in a patient who may have suffered inhalation injury might indicate hypoxia, and this should be assessed by the nurse first. Administration of morphine may be indicated if the nurse determines that the agitation is caused by pain. Assessing level of consciousness and orientation is also appropriate but not as essential as determining whether the patient is hypoxemic. Reassurance is not helpful to reduce agitation in a hypoxemic patient.

Which of these patients is most appropriate for the burn unit charge nurse to assign to an RN staff nurse who has floated from the hospital medical unit? a.A 63-year-old patient who has blebs under an autograft on the thigh and has an order for bleb aspiration b.A 45-year-old patient who has just come back to the unit after having a cultured epithelial autograft to the chest c.A 60-year-old patient who has twice-daily burn debridements and dressing changes to partial-thickness facial burns d.A 34-year-old patient who has a weight loss of 15% from admission and requires enteral feedings and parenteral nutrition (PN)

ANS: D An RN from a medical unit would be familiar with malnutrition and with administration and evaluation of response to enteral feedings and PN. The other patients require burn assessment and care that is more appropriate for staff who regularly care for burned patients.

Which of these laboratory results requires the most rapid action by the nurse who is caring for a patient who suffered a large burn 48 hours ago? a.Hct 52% b.BUN 36 mg/dL c.Serum sodium 146 mEq/L d.Serum potassium 6.2 mEq/L

ANS: D Hyperkalemia can lead to fatal bradycardia and indicates that the patient requires cardiac monitoring and immediate treatment to lower the potassium level. The other laboratory values also are abnormal and require changes in treatment, but they are not as immediately life threatening as the elevated potassium level.

An older adult patient with a squamous cell carcinoma (SCC) on the lower arm has a Mohs procedure in the dermatology clinic. Which nursing action will be included in the postoperative plan of care? a.Describe the use of topical fluorouracil on the incision. b.Teach how to use sterile technique to clean the suture line. c.Schedule daily appointments for wet-to-dry dressing changes. d.Teach about the use of cold packs to reduce bruising and swelling.

ANS: D Application of cold packs to the incision after the surgery will help decrease bruising and swelling at the site. Since the Mohs procedure results in complete excision of the lesion, topical fluorouracil is not needed after surgery. After the Mohs procedure the edges of the wound can be left open to heal or the edges can be approximated and sutured together. The suture line can be cleaned with tap water. No debridement with wet-to-dry dressings is indicated.

Which action will the nurse include in the plan of care for a patient in the rehabilitation phase after a burn injury to the right arm and chest? a.Keep the right arm in a position of comfort. b.Avoid the use of sustained-release narcotics. c.Teach about the purpose of tetanus immunization. d.Apply water-based cream to burned areas frequently.

ANS: D Application of water-based emollients will moisturize new skin and decrease flakiness and itching. To avoid contractures, the joints of the right arm should be positioned in an extended position, which is not the position of comfort. Patients may need to continue the use of opioids during rehabilitation. Tetanus immunization would have been given during the emergent phase of the burn injury.

A nurse develops a teaching plan for a patient diagnosed with basal cell carcinoma (BCC). Which information should the nurse include in the teaching plan? a.Treatment plans include watchful waiting. b.Screening for metastasis will be important. c.Low dose systemic chemotherapy is used to treat BCC. d.Minimizing sun exposure will reduce risk for future BCC.

ANS: D BCC is frequently associated with sun exposure and preventive measures should be taken for future sun exposure. BCC spreads locally, and does not metastasize to distant tissues. Since BCC can cause local tissue destruction, treatment is indicated. Local (not systemic) chemotherapy may be used to treat BCC.

Which assessment information documented in a patients chart indicates that the nurse may need to continue to monitor the skin condition of an 82-year-old patient admitted with bacterial pneumonia? a.Scattered macular brown areas on extremities b.Skin brown and wrinkled, skin tenting on forearm c.Longitudinal nail bed ridges noted, sparse scalp hair d.Skin moist and intact; states history of allergic rashes

ANS: D Because the patient will be receiving antibiotics, the nurse should monitor the patient for the presence of an allergic rash. The assessment data in the other response would be normal for an elderly patient.

What is the best method to prevent the spread of infection when the nurse is changing the dressing over a wound infected with Staphylococcus aureus? a.Change the dressing using sterile gloves. b.Soak the dressing in sterile normal saline. c.Apply antibiotic ointment over the wound. d.Wash hands and properly dispose of soiled dressings.

ANS: D Careful hand washing and the safe disposal of soiled dressings are the best means of preventing the spread of skin problems. Sterile glove and sterile saline use during wound care will not necessarily prevent spread of infection. Applying antibiotic ointment will treat the bacteria but not necessarily prevent the spread of infection.

When admitting a patient with stage III pressure ulcers on both heels, which information obtained by the nurse will have the most impact on wound healing? a.The patient states that the ulcers are very painful. b.The patient has had the heel ulcers for the last 6 months. c.The patient has several old incisions that have formed keloids. d.The patient takes corticosteroids daily for rheumatoid arthritis.

ANS: D Chronic corticosteroid use will interfere with wound healing. The persistence of the ulcers over the last 6 months is a concern, but changes in care may be effective in promoting healing. Keloids are not disabling or painful, although the cosmetic effects may be distressing for some patients. Actions to reduce the patients pain will be implemented, but pain does not impact directly on wound healing.

A patient with rheumatoid arthritis has been taking corticosteroids for 11 months. Which nursing action is most likely to detect early signs of infection in this patient? a.Monitor white blood cell count. b.Check the skin for areas of redness. c.Check the temperature every 2 hours. d.Ask about fatigue or feelings of malaise.

ANS: D Common clinical manifestations of inflammation and infection are frequently not present when patients receive immunosuppressive medications. The earliest manifestation of an infection may be just not feeling well.

Which nursing action is most likely to detect early signs of infection in a patient who is taking immunosuppressive medications? a.Monitor white blood cell count. b.Check the skin for areas of redness. c.Check the temperature every 2 hours. d.Ask about fatigue or feelings of malaise.

ANS: D Common clinical manifestations of inflammation and infection are frequently not present when patients receive immunosuppressive medications. The earliest manifestation of an infection may be just not feeling well.

The nurse has just received change-of-shift report about the following four patients. Which patient will the nurse assess first? a.The patient who has multiple black wounds on the feet and ankles. b.The newly admitted patient with a stage IV pressure ulcer on the coccyx. c.The patient who needs to be medicated with multiple analgesics before a scheduled dressing change. d.The patient who has been receiving immunosuppressant medications and has a temperature of 102 F.

ANS: D Even a low fever in an immunosuppressed patient is a sign of serious infection and should be treated immediately with cultures and rapid initiation of antibiotic therapy. The nurse should assess the other patients as soon as possible after assessing and implementing appropriate care for the immunosuppressed patient.

Esomeprazole (Nexium) is prescribed for a patient who incurred extensive burn injuries 5 days ago. Which nursing assessment would best evaluate the effectiveness of the medication? a.Bowel sounds b.Stool frequency c.Abdominal distention d.Stools for occult blood

ANS: D H2 blockers and proton pump inhibitors are given to prevent Curlings ulcer in the patient who has suffered burn injuries. Proton pump inhibitors usually do not affect bowel sounds, stool frequency, or appetite.

The nurse could delegate care of which patient to a licensed practical/vocational nurse (LPN/LVN)? a.The patient who has increased tenderness and swelling around a leg wound b.The patient who was just admitted after suturing of a full-thickness arm wound c.The patient who needs teaching about home care for a draining abdominal wound d.The patient who requires a hydrocolloid dressing change for a stage III sacral ulcer

ANS: D LPN/LVN education and scope of practice include sterile dressing changes for stable patients. Initial wound assessments, patient teaching, and evaluation for possible poor wound healing or infection should be done by the registered nurse (RN).

The nurse will perform which action when doing a wet-to-dry dressing change on a patients stage III sacral pressure ulcer? a.Soak the old dressings with sterile saline 30 minutes before removing them. b.Pour sterile saline onto the new dry dressings after the wound has been packed. c.Apply antimicrobial ointment before repacking the wound with moist dressings. d.Administer the ordered PRN hydrocodone (Lortab) 30 minutes before the dressing change.

ANS: D Mechanical debridement with wet-to-dry dressings is painful, and patients should receive pain medications before the dressing change begins. The new dressings are moistened with saline before being applied to the wound. Soaking the old dressings before removing them will eliminate the wound debridement that is the purpose of this type of dressing. Application of antimicrobial ointments is not indicated for a wet-to-dry dressing.

A 24-year-old patient who is receiving antibiotics for an infected leg wound has a temperature of 101.8 F (38.7 C). Which action by the nurse is most appropriate? a.Apply a cooling blanket. b.Notify the health care provider. c.Give the prescribed PRN aspirin (Ascriptin) 650 mg. d.Check the patients oral temperature again in 4 hours.

ANS: D Mild to moderate temperature elevations (less than 103 F) do not harm the young adult patient and may benefit host defense mechanisms. The nurse should continue to monitor the temperature. Antipyretics are not indicated unless the patient is complaining of fever-related symptoms. There is no need to notify the patients health care provider or to use a cooling blanket for a moderate temperature elevation.

The nurse is reviewing the medication administration record (MAR) on a patient with partial-thickness burns. Which medication is best for the nurse to administer before scheduled wound debridement? a.Ketorolac (Toradol) b.Lorazepam (Ativan) c.Gabapentin (Neurontin) d.Hydromorphone (Dilaudid)

ANS: D Opioid pain medications are the best choice for pain control. The other medications are used as adjuvants to enhance the effects of opioids.

The nurse assesses a patient who has just arrived in the postanesthesia recovery area (PACU) after a blepharoplasty. Which assessment data should be reported to the surgeon immediately? a.The patient complains of incisional pain. b.The patients heart rate is 110 beats/minute. c.The patient is unable to detect when the eyelids are touched. d.The skin around the incision is pale and cold when palpated.

ANS: D Pale, cool skin indicates a possible decrease in circulation, so the surgeon should be notified immediately. The other assessment data indicate a need for ongoing assessment or nursing action. A heart rate of 110 beats/minute may be related to the stress associated with surgery. Assessment of other vital signs and continued monitoring are appropriate. Because local anesthesia would be used for the procedure, numbness of the incisional area is expected immediately after surgery. The nurse should monitor for return of feeling.

The nurse notes these abnormalities on the skin of a 95-year-old patient who is being admitted to an assisted living facility. Which abnormality is the priority to discuss immediately with the health care provider? a.Several dry, scaly patches on the face b.Numerous varicosities noted on both legs c.Dilation of small blood vessels on the face d.Petechiae present on the chest and abdomen

ANS: D Petechiae are caused by pinpoint hemorrhages and are associated with a variety of serious disorders such as meningitis and coagulopathies. The nurse should contact the patients health care provider about this finding for further diagnostic follow-up. The other skin changes are associated with aging. Although the other changes also will require ongoing monitoring or intervention by the nurse, they do not indicate a need for urgent action.

After the home health nurse teaches a patients family member about how to care for a sacral pressure ulcer, which finding indicates that additional teaching is needed? a.The family member uses a lift sheet to reposition the patient. b.The family member uses clean tap water to clean the wound. c.The family member places contaminated dressings in a plastic grocery bag. d.The family member dries the wound using a hair dryer set on a low setting.

ANS: D Pressure ulcers need to be kept moist to facilitate wound healing. The other actions indicate a good understanding of pressure ulcer care.

A patient with an enlarging, irregular mole that is 6 mm in diameter is scheduled for outpatient treatment. The nurse should plan on teaching the patient about a.curettage. b.cryosurgery. c.punch biopsy. d.surgical excision.

ANS: D The description of the mole is consistent with malignancy, so excision and biopsy are indicated. Curettage and cryosurgery are not used if malignancy is suspected. A punch biopsy would not be done for a lesion greater than 5 mm in diameter.

A patient with an enlarging, irregular mole that is 7 mm in diameter is scheduled for outpatient treatment. The nurse should plan to prepare the patient for which procedure? a.Curettage b.Cryosurgery c.Punch biopsy d.Surgical excision

ANS: D The description of the mole is consistent with malignancy, so excision and biopsy are indicated. Curettage and cryosurgery are not used if malignancy is suspected. A punch biopsy would not be done for a lesion greater than 5 mm in diameter.

A young male patient who is a paraplegic has a stage II sacral pressure ulcer and is being cared for at home by his mother. To prevent further tissue damage, what instructions are most important for the nurse to teach the mother? a. Change the patients bedding frequently. b. Use a hydrocolloid dressing over the ulcer. c. Record the size and appearance of the ulcer weekly. d. Change the patients position at least every 2 hours.

ANS: D The most important intervention is to avoid prolonged pressure on bony prominences by frequent repositioning. The other interventions may also be included in family teaching, but the most important instruction is to change the patients position at least every 2 hours.

A patient who has severe refractory psoriasis on the face, neck, and extremities has quit working and withdrawn from social activities because of the appearance of the lesions. Which action should the nurse take first? a.Discuss the possibility of enrolling in a worker-retraining program. b.Encourage the patient to volunteer to work on community projects. c.Suggest that the patient use cosmetics to cover the psoriatic lesions. d.Ask the patient to describe the impact of psoriasis on quality of life.

ANS: D The nurses initial actions should be to assess the impact of the disease on the patients life and to allow the patient to verbalize feelings about the psoriasis. Depending on the assessment findings, other actions may be appropriate.

A patient with atopic dermatitis has a new prescription for tacrolimus (Protopic). After teaching the patient about the medication, which statement by the patient indicates that further teaching is needed? a.After I apply the medication, I can go ahead and get dressed as usual. b.I will rub the medication gently onto the skin every morning and night. c.I will need to minimize my time in the sun while I am using the Protopic. d.If the medication burns when I apply it, I will wipe it off and call the doctor.

ANS: D The patient should be taught that transient burning at the application site is an expected effect of tacrolimus and that the medication should be left in place. The other statements by the patient are accurate and indicate that patient teaching has been effective.

A patient who was found unconscious in a burning house is brought to the emergency department by ambulance. The nurse notes that the patients skin color is bright red. Which action should the nurse take first? a.Insert two large-bore IV lines. b.Check the patients orientation. c.Assess for singed nasal hair and dark oral mucous membranes. d.Place the patient on 100% oxygen using a non-rebreather mask.

ANS: D The patients history and skin color suggest carbon monoxide poisoning, which should be treated by rapidly starting oxygen at 100%. The other actions can be taken after the action to correct gas exchange.

The nurse notes darker skin pigmentation in the skinfolds of a middle-aged patient who has a body mass index of 40 kg/m2. What is the nurses best action? a.Teach the patient about the treatment of fungal infection. b.Discuss the use of drying agents to minimize infection risk. c.Instruct the patient about the use of mild soap to clean skinfolds. d.Ask the patient about type 2 diabetes or if there is a family history of it.

ANS: D The presence of acanthosis nigricans in skinfolds suggests either having type 2 diabetes or being at an increased risk for it. The description of the patients skin does not indicate problems with fungal infection, poor hygiene, or the need to dry the skinfolds better.

Which information should the nurse include when teaching patients about decreasing the risk for sun damage to the skin? a.Use a sunscreen with an SPF of at least 8 to 10 for adequate protection. b.Water resistant sunscreens will provide good protection when swimming. c.Increase sun exposure by no more than 10 minutes a day to avoid skin damage. d.Try to stay out of the sun between the hours of 10 AM and 2 PM (regular time).

ANS: D The risk for skin damage from the sun is highest with exposure between 10 AM and 2 PM. No sunscreen is completely water resistant. Sunscreens classified as water resistant sunscreens still need to be reapplied after swimming. Sunscreen with an SPF of at least 15 is recommended for people at normal risk for skin cancer. Although gradually increasing sun exposure may decrease the risk for burning, the risk for skin cancer is not decreased.

When examining a homebound patient, the home health nurse notes a musky, sour body odor. Based on this assessment, the most appropriate nursing action is to a.teach the patient to apply a moisturizing body lotion daily. b.ask about use of over-the-counter (OTC) skin medications. c.ask the health care provider about a prescription for a topical antifungal. d.schedule nursing assistive personnel to help with bathing several times weekly.

ANS: D The skin odor indicates that the patients hygiene is poor and that assistance with bathing is needed. Although elderly patients may need moisturizing lotions and should be asked about use of skin medications, the assessment data do not indicate that these are the most appropriate actions. An antifungal would be indicated if the nurse noticed a yeast odor.

Six hours after a thermal burn covering 50% of a patients total body surface area (TBSA), the nurse obtains these data when assessing a patient. What is the priority information to communicate to the health care provider? a.Blood pressure is 94/46 per arterial line. b.Serous exudate is leaking from the burns. c.Cardiac monitor shows a pulse rate of 104. d.Urine output is 20 mL per hour for the past 2 hours.

ANS: D The urine output should be at least 0.5 to 1.0 mL/kg/hr during the emergent phase, when the patient is at great risk for hypovolemic shock. The nurse should notify the health care provider because a higher IV fluid rate is needed. BP during the emergent phase should be greater than 90 systolic, and the pulse rate should be less than 120. Serous exudate from the burns is expected during the emergent phase.

The nurse should plan to use a wet-to-dry dressing for which patient? a.A patient who has a pressure ulcer with pink granulation tissue b.A patient who has a surgical incision with pink, approximated edges c.A patient who has a full-thickness burn filled with dry, black material d.A patient who has a wound with purulent drainage and dry brown areas

ANS: D Wet-to-dry dressings are used when there is minimal eschar to be removed. A full-thickness wound filled with eschar will require interventions such as surgical debridement to remove the necrotic tissue. Wet-to-dry dressings are not needed on approximated surgical incisions. Wet-to-dry dressings are not used on uninfected granulating wounds because of the damage to the granulation tissue.

During the emergent phase of burn care, which assessment will be most useful in determining whether the patient is receiving adequate fluid infusion? a.Check skin turgor. b.Monitor daily weight. c.Assess mucous membranes. d.Measure hourly urine output.

ANS: D When fluid intake is adequate, the urine output will be at least 0.5 to 1 mL/kg/hour. The patients weight is not useful in this situation because of the effects of third spacing and evaporative fluid loss. Mucous membrane assessment and skin turgor also may be used, but they are not as adequate in determining that fluid infusions are maintaining adequate perfusion.

The nurse assesses a circular, flat, reddened lesion about 5 cm in diameter on a middle-aged patients ankle. How should the nurse determine if the lesion is related to intradermal bleeding? a.Elevate the patients leg. b.Press firmly on the lesion. c.Check the temperature of the skin around the lesion. d.Palpate the dorsalis pedis and posterior tibial pulses.

B If the lesion is caused by intradermal or subcutaneous bleeding or a nonvascular cause, the discoloration will remain when direct pressure is applied to the lesion. If the lesion is caused by blood vessel dilation, blanching will occur with direct pressure. The other assessments will assess circulation to the leg, but will not be helpful in determining the etiology of the lesion.


Conjuntos de estudio relacionados

Pre-Lecture Quiz: Chapter 14 Before Conception (Look at numbers 66-75)

View Set

Postulate 3-1, Theorems 3-(1,2,3)

View Set

New Testament Final Exam Vocab Guide

View Set

Karch's PrepU (Pharm) Ch. 14: Antineoplastic Agents

View Set

Assignment: 8. Quiz 2: Animal-like Protists

View Set